lunes, 28 de marzo de 2011

Posible demostración de "otro problema"

2 comentarios:

  1. Creo que he visto una errata en lo escrito, cuatro lineas antes de terminar deberías poner:
    $(x_n)_{n \in \mathbb{N}} \subset \overline{(x_n)}_{n \in \mathbb{N}}$

    Por lo demás creo que si esta probado.

    ResponderEliminar
  2. Muchas gracias Miguel. Eso era lo que queria escribir, pero se me escapó escribir la linea de arriba :)

    ResponderEliminar